Showing $sum_{k=1}^{nm} frac{1}{k} approx sum_{k=1}^{n} frac{1}{k} + sum_{k=1}^{m} frac{1}{k}$












13












$begingroup$


Since $log(nm) = log(n) + log(m)$, and $sum_{k=1}^n frac{1}{k} approx log n$ for large $n$, we would expect that $$sum_{k=1}^{nm} frac{1}{k} approx sum_{k=1}^{n} frac{1}{k} + sum_{k=1}^{m} frac{1}{k}$$



when $n,m$ are large.



I'm wondering if this approximation can be demonstrated through discrete means. That is to say, manipulations of rational fractions and/or elementary number-theoretical considerations, without using the $log n$ approximation.










share|cite|improve this question









$endgroup$








  • 4




    $begingroup$
    There is a constant error in the approximation of $gamma approx 0.577$ as the harmonic numbers are about $H_n approx log n + gamma$ so there are two $gamma$'s on the right but only one on the left. This is still a reasonable idea.
    $endgroup$
    – Ross Millikan
    Jan 1 at 3:25








  • 2




    $begingroup$
    Probably you know this, but just in case you don't: You can using integrals and the Fundamental Theorem of Calculus to define logs. Define $L(x) = int_1^x frac{1}{t} dt$. Then you can check: $L'(x) = 1/x$, $L(ax) = L(a) + L(x)$, and $L(x^p) = pL(x)$. Your question is the series version of showing that $int_1^{ax} frac{1}{t} dt = int_1^a frac{1}{t} dt + int_1^x frac{1}{t}dt$.
    $endgroup$
    – JavaMan
    Jan 1 at 4:02
















13












$begingroup$


Since $log(nm) = log(n) + log(m)$, and $sum_{k=1}^n frac{1}{k} approx log n$ for large $n$, we would expect that $$sum_{k=1}^{nm} frac{1}{k} approx sum_{k=1}^{n} frac{1}{k} + sum_{k=1}^{m} frac{1}{k}$$



when $n,m$ are large.



I'm wondering if this approximation can be demonstrated through discrete means. That is to say, manipulations of rational fractions and/or elementary number-theoretical considerations, without using the $log n$ approximation.










share|cite|improve this question









$endgroup$








  • 4




    $begingroup$
    There is a constant error in the approximation of $gamma approx 0.577$ as the harmonic numbers are about $H_n approx log n + gamma$ so there are two $gamma$'s on the right but only one on the left. This is still a reasonable idea.
    $endgroup$
    – Ross Millikan
    Jan 1 at 3:25








  • 2




    $begingroup$
    Probably you know this, but just in case you don't: You can using integrals and the Fundamental Theorem of Calculus to define logs. Define $L(x) = int_1^x frac{1}{t} dt$. Then you can check: $L'(x) = 1/x$, $L(ax) = L(a) + L(x)$, and $L(x^p) = pL(x)$. Your question is the series version of showing that $int_1^{ax} frac{1}{t} dt = int_1^a frac{1}{t} dt + int_1^x frac{1}{t}dt$.
    $endgroup$
    – JavaMan
    Jan 1 at 4:02














13












13








13


4



$begingroup$


Since $log(nm) = log(n) + log(m)$, and $sum_{k=1}^n frac{1}{k} approx log n$ for large $n$, we would expect that $$sum_{k=1}^{nm} frac{1}{k} approx sum_{k=1}^{n} frac{1}{k} + sum_{k=1}^{m} frac{1}{k}$$



when $n,m$ are large.



I'm wondering if this approximation can be demonstrated through discrete means. That is to say, manipulations of rational fractions and/or elementary number-theoretical considerations, without using the $log n$ approximation.










share|cite|improve this question









$endgroup$




Since $log(nm) = log(n) + log(m)$, and $sum_{k=1}^n frac{1}{k} approx log n$ for large $n$, we would expect that $$sum_{k=1}^{nm} frac{1}{k} approx sum_{k=1}^{n} frac{1}{k} + sum_{k=1}^{m} frac{1}{k}$$



when $n,m$ are large.



I'm wondering if this approximation can be demonstrated through discrete means. That is to say, manipulations of rational fractions and/or elementary number-theoretical considerations, without using the $log n$ approximation.







sequences-and-series harmonic-numbers






share|cite|improve this question













share|cite|improve this question











share|cite|improve this question




share|cite|improve this question










asked Jan 1 at 3:05









MathematicsStudent1122MathematicsStudent1122

8,62322467




8,62322467








  • 4




    $begingroup$
    There is a constant error in the approximation of $gamma approx 0.577$ as the harmonic numbers are about $H_n approx log n + gamma$ so there are two $gamma$'s on the right but only one on the left. This is still a reasonable idea.
    $endgroup$
    – Ross Millikan
    Jan 1 at 3:25








  • 2




    $begingroup$
    Probably you know this, but just in case you don't: You can using integrals and the Fundamental Theorem of Calculus to define logs. Define $L(x) = int_1^x frac{1}{t} dt$. Then you can check: $L'(x) = 1/x$, $L(ax) = L(a) + L(x)$, and $L(x^p) = pL(x)$. Your question is the series version of showing that $int_1^{ax} frac{1}{t} dt = int_1^a frac{1}{t} dt + int_1^x frac{1}{t}dt$.
    $endgroup$
    – JavaMan
    Jan 1 at 4:02














  • 4




    $begingroup$
    There is a constant error in the approximation of $gamma approx 0.577$ as the harmonic numbers are about $H_n approx log n + gamma$ so there are two $gamma$'s on the right but only one on the left. This is still a reasonable idea.
    $endgroup$
    – Ross Millikan
    Jan 1 at 3:25








  • 2




    $begingroup$
    Probably you know this, but just in case you don't: You can using integrals and the Fundamental Theorem of Calculus to define logs. Define $L(x) = int_1^x frac{1}{t} dt$. Then you can check: $L'(x) = 1/x$, $L(ax) = L(a) + L(x)$, and $L(x^p) = pL(x)$. Your question is the series version of showing that $int_1^{ax} frac{1}{t} dt = int_1^a frac{1}{t} dt + int_1^x frac{1}{t}dt$.
    $endgroup$
    – JavaMan
    Jan 1 at 4:02








4




4




$begingroup$
There is a constant error in the approximation of $gamma approx 0.577$ as the harmonic numbers are about $H_n approx log n + gamma$ so there are two $gamma$'s on the right but only one on the left. This is still a reasonable idea.
$endgroup$
– Ross Millikan
Jan 1 at 3:25






$begingroup$
There is a constant error in the approximation of $gamma approx 0.577$ as the harmonic numbers are about $H_n approx log n + gamma$ so there are two $gamma$'s on the right but only one on the left. This is still a reasonable idea.
$endgroup$
– Ross Millikan
Jan 1 at 3:25






2




2




$begingroup$
Probably you know this, but just in case you don't: You can using integrals and the Fundamental Theorem of Calculus to define logs. Define $L(x) = int_1^x frac{1}{t} dt$. Then you can check: $L'(x) = 1/x$, $L(ax) = L(a) + L(x)$, and $L(x^p) = pL(x)$. Your question is the series version of showing that $int_1^{ax} frac{1}{t} dt = int_1^a frac{1}{t} dt + int_1^x frac{1}{t}dt$.
$endgroup$
– JavaMan
Jan 1 at 4:02




$begingroup$
Probably you know this, but just in case you don't: You can using integrals and the Fundamental Theorem of Calculus to define logs. Define $L(x) = int_1^x frac{1}{t} dt$. Then you can check: $L'(x) = 1/x$, $L(ax) = L(a) + L(x)$, and $L(x^p) = pL(x)$. Your question is the series version of showing that $int_1^{ax} frac{1}{t} dt = int_1^a frac{1}{t} dt + int_1^x frac{1}{t}dt$.
$endgroup$
– JavaMan
Jan 1 at 4:02










4 Answers
4






active

oldest

votes


















7












$begingroup$

This is a nice little problem, and thankfully it has a simple solution.



Assume that $m$ and $n$ are large natural numbers. Consider the following inequalities:



$$
sum_{k=cn+1}^{(c+1)n}frac{1}{(c+1)n}leqsum_{k=cn+1}^{(c+1)n}frac{1}{k} leqsum_{k=cn+1}^{(c+1)n}frac{1}{cn+1}
$$

This means that we may estimate the middle sum with the lower sum with an error no greater than the following (using the fact that $n$ is large):
$$
sum_{k=cn+1}^{(c+1)n}frac{n-1}{(c+1)n(nc+1)} approx sum_{k=cn+1}^{(c+1)n}frac{1}{c(c+1)n} = frac{1}{c(c+1)}
$$

Thus we have the following approximate upper bound (using the fact that $m$ is large):
$$
sum_{k=1}^{nm}frac{1}{k} = sum_{k=1}^{n}frac{1}{k} + sum_{c=1}^{m-1}sum_{k=cn+1}^{(c+1)n}frac{1}{k} lesssimsum_{k=1}^nfrac{1}{k}+ sum_{c=1}^{m-1}left[frac{1}{c+1}+frac{1}{c(c+1)}right]approxsum_{k=1}^nfrac{1}{k}+sum_{k=1}^mfrac{1}{k}
$$

And we have the following lower bound:
$$
sum_{k=1}^{nm}frac{1}{k} = sum_{k=1}^{n}frac{1}{k} + sum_{c=1}^{m-1}sum_{k=cn+1}^{(c+1)n}frac{1}{k} geqsum_{k=1}^nfrac{1}{k}+ sum_{c=1}^{m-1}frac{1}{c+1} =left[sum_{k=1}^nfrac{1}{k}+sum_{k=1}^mfrac{1}{k}right]-1
$$






share|cite|improve this answer











$endgroup$









  • 1




    $begingroup$
    As an aside, all of the $approx$'s and the $lesssim$ can be replaced with $leq$'s. So this argument is fully rigorous, but it is also not sharp; one of the other answers does better with the upper bound.
    $endgroup$
    – InequalitiesEverywhere
    Jan 1 at 4:08



















4












$begingroup$

Let $$S = sum_{k=1}^n frac{1}k + sum_{k=1}^m frac{1}k.$$
Note that we can write $sum_{k=1}^{nm} frac{1}k$ as
$$begin{align*}
1 + cdots + frac{1}n \
frac1{n+1} + cdots + frac{1}{2n} \
cdots \
frac{1}{n(m-1)} + cdots + frac{1}{nm}
end{align*} $$

Label the rows of the above table from $1$ upto $m$. Note that all the numbers in the $j$th row are $ge frac{1}{jn}$ so the sum of the numbers in the $j$th row is at least $n cdot frac{1}{jn} = frac{1}j$. Summing this from $j=2$ to $m$ gives us
$$ sum_{k=1}^{nm} frac{1}k ge left(sum_{k=1}^n frac{1}k + sum_{j=1}^m frac{1}j right) - 1 = S-1.$$
Similarly, each entry in the $j$th row is at most $frac{1}{(j-1)n}$ so summing from $j=2$ to $m$ gives
$$ sum_{k=1}^{nm} frac{1}k le left(sum_{k=1}^n frac{1}k + sum_{j=1}^m frac{1}j right) - frac{1}m = S-frac{1}m.$$
Therefore,
$$ frac{1}m le sum_{k=1}^n frac{1}k + sum_{k=1}^m frac{1}k - sum_{k=1}^{nm} frac{1}k le 1. $$






share|cite|improve this answer









$endgroup$





















    2












    $begingroup$

    Avoiding $log$s we still have $H_N=sum_{k=1}^{N}frac{1}{k}=int_{0}^{1}frac{x^N-1}{x-1}$ and



    $$ H_{NM}-H_{N}-H_{M} = int_{0}^{1}frac{(x^N-1)(x^M-1)}{x-1},dxleq 0, $$
    whose absolute value can be bounded (due to the Cauchy-Schwarz inequality) by
    $$ sqrt{int_{0}^{1}frac{(1-x^N)^2}{1-x},dx int_{0}^{1}frac{(1-x^M)^2}{1-x},dx}=sqrt{(2H_N-H_{2N})(2H_M-H_{2M})}. $$
    On the other hand
    $$ H_{2N}-H_N = sum_{k=1}^{2N}frac{(-1)^{k+1}}{k} $$
    is a partial sum for a convergent (to $log 2$) series, hence
    $$ H_N+H_M-sqrt{H_N H_M}leq H_{NM} leq H_N+H_M $$
    holds for any large $N,M$.






    share|cite|improve this answer









    $endgroup$





















      1












      $begingroup$

      Be the sucession ${c_n}_{ninmathbb{N}}$ as:
      begin{equation}
      c_n = sum_{k=1}^n left(frac{1}{k}right) - ln(n)
      end{equation}

      We can notice that:
      begin{eqnarray*}
      c_{n+1}-c_n &=& left[sum_{k=1}^{n+1} left(frac{1}{k}right) - ln(n+1)right]
      -left[sum_{k=1}^{n} left(frac{1}{k}right) - ln(n)right] \
      c_{n+1}-c_n &=& frac{1}{n+1} - ln(n+1) + ln(n) \
      c_{n+1}-c_n &=& frac{1}{n+1} - left[ln(n+1) - ln(n)right] \
      c_{n+1}-c_n &=& frac{1}{n+1} - int_{n}^{n+1} frac{1}{x} dx \
      end{eqnarray*}

      But if we graph $displaystyle y = frac{1}{x}$ and we draw rectangles as in the figure:



      Figure



      We notice that the area of the rectangle between $x=n$ and $x=n+1$ is $displaystylefrac{1}{n+1}$ and it is less than $displaystyleint_{n}^{n+1} frac{1}{x} dx$. Then $forall nin mathbb{N}$:



      begin{equation}
      c_{n+1}-c_n = frac{1}{n+1} - int_{n}^{n+1} frac{1}{x} dx < 0
      end{equation}



      Thus, the sucession ${c_n}_{ninmathbb{N}}$ is monotonically decreasing and $c_{n+1} < c_n < cdots < c_2 < c_1 = 1$, then $lim_{ntoinfty}c_n=gamma$ exists and it is less than 1, it should be noted that $lim_{ntoinfty}c_n$ is the Euler-Mascheroni constant $gamma$.



      Be the sucession $leftlbrace b_n rightrbrace_{ninmathbb{N}}$ such that $b_n=nm$ with $minmathbb{N}$, the sucession $c_n$ and its subsucession $c_{b_n}$ converge to the same number:
      begin{eqnarray}
      lim_{ntoinfty}c_{b_n} &=& lim_{ntoinfty}c_n \
      lim_{ntoinfty}sum_{k=1}^{nm} left(frac{1}{k}right) - ln(nm) &=& lim_{ntoinfty}sum_{k=1}^n left(frac{1}{k}right) - ln(n) \
      lim_{ntoinfty}sum_{k=1}^{nm} left(frac{1}{k}right) - ln(n) - ln(m) &=& lim_{ntoinfty}sum_{k=1}^n left(frac{1}{k}right) - ln(n) \
      lim_{ntoinfty}sum_{k=1}^{nm} left(frac{1}{k}right) -sum_{k=1}^n left(frac{1}{k}right) &=& ln(m) \
      lim_{ntoinfty}sum_{k=1}^{nm} left(frac{1}{k}right) -sum_{k=1}^n left(frac{1}{k}right) -sum_{k=1}^{m} left(frac{1}{k}right) &=& ln(m) -sum_{k=1}^{m} left(frac{1}{k}right) \
      lim_{mtoinfty}lim_{ntoinfty}sum_{k=1}^{nm} left(frac{1}{k}right) -sum_{k=1}^n left(frac{1}{k}right) -sum_{k=1}^{m} left(frac{1}{k}right)
      &=&
      lim_{mtoinfty}ln(m) -sum_{k=1}^{m} left(frac{1}{k}right) \
      lim_{mtoinfty}lim_{ntoinfty}sum_{k=1}^{nm} left(frac{1}{k}right) -sum_{k=1}^n left(frac{1}{k}right) -sum_{k=1}^{m} left(frac{1}{k}right)
      &=&
      gamma < 1
      end{eqnarray}



      Then for $n,m$ very big:
      begin{eqnarray}
      sum_{k=1}^{nm} left(frac{1}{k}right) -sum_{k=1}^n left(frac{1}{k}right) -sum_{k=1}^{m} left(frac{1}{k}right) &approx& gamma \
      sum_{k=1}^{nm} left(frac{1}{k}right) &approx& gamma + sum_{k=1}^n left(frac{1}{k}right) + sum_{k=1}^{m} left(frac{1}{k}right) \
      sum_{k=1}^{nm} left(frac{1}{k}right) &approx& sum_{k=1}^n left(frac{1}{k}right) + sum_{k=1}^{m} left(frac{1}{k}right)
      end{eqnarray}

      Because $displaystyle gamma << sum_{k=1}^n left(frac{1}{k}right) + sum_{k=1}^{m} left(frac{1}{k}right)$.
      Part of this demonstration is from the book "Euler: The master of us all" [1].





      References:
      Dunham, William, Euler: The master of us all, The Dolciani Mathematical Expositions. 22. Washington, DC: The Mathematical Association of America. xxviii, 185 p. (1999). ZBL0951.01012.






      share|cite|improve this answer











      $endgroup$













        Your Answer





        StackExchange.ifUsing("editor", function () {
        return StackExchange.using("mathjaxEditing", function () {
        StackExchange.MarkdownEditor.creationCallbacks.add(function (editor, postfix) {
        StackExchange.mathjaxEditing.prepareWmdForMathJax(editor, postfix, [["$", "$"], ["\\(","\\)"]]);
        });
        });
        }, "mathjax-editing");

        StackExchange.ready(function() {
        var channelOptions = {
        tags: "".split(" "),
        id: "69"
        };
        initTagRenderer("".split(" "), "".split(" "), channelOptions);

        StackExchange.using("externalEditor", function() {
        // Have to fire editor after snippets, if snippets enabled
        if (StackExchange.settings.snippets.snippetsEnabled) {
        StackExchange.using("snippets", function() {
        createEditor();
        });
        }
        else {
        createEditor();
        }
        });

        function createEditor() {
        StackExchange.prepareEditor({
        heartbeatType: 'answer',
        autoActivateHeartbeat: false,
        convertImagesToLinks: true,
        noModals: true,
        showLowRepImageUploadWarning: true,
        reputationToPostImages: 10,
        bindNavPrevention: true,
        postfix: "",
        imageUploader: {
        brandingHtml: "Powered by u003ca class="icon-imgur-white" href="https://imgur.com/"u003eu003c/au003e",
        contentPolicyHtml: "User contributions licensed under u003ca href="https://creativecommons.org/licenses/by-sa/3.0/"u003ecc by-sa 3.0 with attribution requiredu003c/au003e u003ca href="https://stackoverflow.com/legal/content-policy"u003e(content policy)u003c/au003e",
        allowUrls: true
        },
        noCode: true, onDemand: true,
        discardSelector: ".discard-answer"
        ,immediatelyShowMarkdownHelp:true
        });


        }
        });














        draft saved

        draft discarded


















        StackExchange.ready(
        function () {
        StackExchange.openid.initPostLogin('.new-post-login', 'https%3a%2f%2fmath.stackexchange.com%2fquestions%2f3058192%2fshowing-sum-k-1nm-frac1k-approx-sum-k-1n-frac1k-sum%23new-answer', 'question_page');
        }
        );

        Post as a guest















        Required, but never shown

























        4 Answers
        4






        active

        oldest

        votes








        4 Answers
        4






        active

        oldest

        votes









        active

        oldest

        votes






        active

        oldest

        votes









        7












        $begingroup$

        This is a nice little problem, and thankfully it has a simple solution.



        Assume that $m$ and $n$ are large natural numbers. Consider the following inequalities:



        $$
        sum_{k=cn+1}^{(c+1)n}frac{1}{(c+1)n}leqsum_{k=cn+1}^{(c+1)n}frac{1}{k} leqsum_{k=cn+1}^{(c+1)n}frac{1}{cn+1}
        $$

        This means that we may estimate the middle sum with the lower sum with an error no greater than the following (using the fact that $n$ is large):
        $$
        sum_{k=cn+1}^{(c+1)n}frac{n-1}{(c+1)n(nc+1)} approx sum_{k=cn+1}^{(c+1)n}frac{1}{c(c+1)n} = frac{1}{c(c+1)}
        $$

        Thus we have the following approximate upper bound (using the fact that $m$ is large):
        $$
        sum_{k=1}^{nm}frac{1}{k} = sum_{k=1}^{n}frac{1}{k} + sum_{c=1}^{m-1}sum_{k=cn+1}^{(c+1)n}frac{1}{k} lesssimsum_{k=1}^nfrac{1}{k}+ sum_{c=1}^{m-1}left[frac{1}{c+1}+frac{1}{c(c+1)}right]approxsum_{k=1}^nfrac{1}{k}+sum_{k=1}^mfrac{1}{k}
        $$

        And we have the following lower bound:
        $$
        sum_{k=1}^{nm}frac{1}{k} = sum_{k=1}^{n}frac{1}{k} + sum_{c=1}^{m-1}sum_{k=cn+1}^{(c+1)n}frac{1}{k} geqsum_{k=1}^nfrac{1}{k}+ sum_{c=1}^{m-1}frac{1}{c+1} =left[sum_{k=1}^nfrac{1}{k}+sum_{k=1}^mfrac{1}{k}right]-1
        $$






        share|cite|improve this answer











        $endgroup$









        • 1




          $begingroup$
          As an aside, all of the $approx$'s and the $lesssim$ can be replaced with $leq$'s. So this argument is fully rigorous, but it is also not sharp; one of the other answers does better with the upper bound.
          $endgroup$
          – InequalitiesEverywhere
          Jan 1 at 4:08
















        7












        $begingroup$

        This is a nice little problem, and thankfully it has a simple solution.



        Assume that $m$ and $n$ are large natural numbers. Consider the following inequalities:



        $$
        sum_{k=cn+1}^{(c+1)n}frac{1}{(c+1)n}leqsum_{k=cn+1}^{(c+1)n}frac{1}{k} leqsum_{k=cn+1}^{(c+1)n}frac{1}{cn+1}
        $$

        This means that we may estimate the middle sum with the lower sum with an error no greater than the following (using the fact that $n$ is large):
        $$
        sum_{k=cn+1}^{(c+1)n}frac{n-1}{(c+1)n(nc+1)} approx sum_{k=cn+1}^{(c+1)n}frac{1}{c(c+1)n} = frac{1}{c(c+1)}
        $$

        Thus we have the following approximate upper bound (using the fact that $m$ is large):
        $$
        sum_{k=1}^{nm}frac{1}{k} = sum_{k=1}^{n}frac{1}{k} + sum_{c=1}^{m-1}sum_{k=cn+1}^{(c+1)n}frac{1}{k} lesssimsum_{k=1}^nfrac{1}{k}+ sum_{c=1}^{m-1}left[frac{1}{c+1}+frac{1}{c(c+1)}right]approxsum_{k=1}^nfrac{1}{k}+sum_{k=1}^mfrac{1}{k}
        $$

        And we have the following lower bound:
        $$
        sum_{k=1}^{nm}frac{1}{k} = sum_{k=1}^{n}frac{1}{k} + sum_{c=1}^{m-1}sum_{k=cn+1}^{(c+1)n}frac{1}{k} geqsum_{k=1}^nfrac{1}{k}+ sum_{c=1}^{m-1}frac{1}{c+1} =left[sum_{k=1}^nfrac{1}{k}+sum_{k=1}^mfrac{1}{k}right]-1
        $$






        share|cite|improve this answer











        $endgroup$









        • 1




          $begingroup$
          As an aside, all of the $approx$'s and the $lesssim$ can be replaced with $leq$'s. So this argument is fully rigorous, but it is also not sharp; one of the other answers does better with the upper bound.
          $endgroup$
          – InequalitiesEverywhere
          Jan 1 at 4:08














        7












        7








        7





        $begingroup$

        This is a nice little problem, and thankfully it has a simple solution.



        Assume that $m$ and $n$ are large natural numbers. Consider the following inequalities:



        $$
        sum_{k=cn+1}^{(c+1)n}frac{1}{(c+1)n}leqsum_{k=cn+1}^{(c+1)n}frac{1}{k} leqsum_{k=cn+1}^{(c+1)n}frac{1}{cn+1}
        $$

        This means that we may estimate the middle sum with the lower sum with an error no greater than the following (using the fact that $n$ is large):
        $$
        sum_{k=cn+1}^{(c+1)n}frac{n-1}{(c+1)n(nc+1)} approx sum_{k=cn+1}^{(c+1)n}frac{1}{c(c+1)n} = frac{1}{c(c+1)}
        $$

        Thus we have the following approximate upper bound (using the fact that $m$ is large):
        $$
        sum_{k=1}^{nm}frac{1}{k} = sum_{k=1}^{n}frac{1}{k} + sum_{c=1}^{m-1}sum_{k=cn+1}^{(c+1)n}frac{1}{k} lesssimsum_{k=1}^nfrac{1}{k}+ sum_{c=1}^{m-1}left[frac{1}{c+1}+frac{1}{c(c+1)}right]approxsum_{k=1}^nfrac{1}{k}+sum_{k=1}^mfrac{1}{k}
        $$

        And we have the following lower bound:
        $$
        sum_{k=1}^{nm}frac{1}{k} = sum_{k=1}^{n}frac{1}{k} + sum_{c=1}^{m-1}sum_{k=cn+1}^{(c+1)n}frac{1}{k} geqsum_{k=1}^nfrac{1}{k}+ sum_{c=1}^{m-1}frac{1}{c+1} =left[sum_{k=1}^nfrac{1}{k}+sum_{k=1}^mfrac{1}{k}right]-1
        $$






        share|cite|improve this answer











        $endgroup$



        This is a nice little problem, and thankfully it has a simple solution.



        Assume that $m$ and $n$ are large natural numbers. Consider the following inequalities:



        $$
        sum_{k=cn+1}^{(c+1)n}frac{1}{(c+1)n}leqsum_{k=cn+1}^{(c+1)n}frac{1}{k} leqsum_{k=cn+1}^{(c+1)n}frac{1}{cn+1}
        $$

        This means that we may estimate the middle sum with the lower sum with an error no greater than the following (using the fact that $n$ is large):
        $$
        sum_{k=cn+1}^{(c+1)n}frac{n-1}{(c+1)n(nc+1)} approx sum_{k=cn+1}^{(c+1)n}frac{1}{c(c+1)n} = frac{1}{c(c+1)}
        $$

        Thus we have the following approximate upper bound (using the fact that $m$ is large):
        $$
        sum_{k=1}^{nm}frac{1}{k} = sum_{k=1}^{n}frac{1}{k} + sum_{c=1}^{m-1}sum_{k=cn+1}^{(c+1)n}frac{1}{k} lesssimsum_{k=1}^nfrac{1}{k}+ sum_{c=1}^{m-1}left[frac{1}{c+1}+frac{1}{c(c+1)}right]approxsum_{k=1}^nfrac{1}{k}+sum_{k=1}^mfrac{1}{k}
        $$

        And we have the following lower bound:
        $$
        sum_{k=1}^{nm}frac{1}{k} = sum_{k=1}^{n}frac{1}{k} + sum_{c=1}^{m-1}sum_{k=cn+1}^{(c+1)n}frac{1}{k} geqsum_{k=1}^nfrac{1}{k}+ sum_{c=1}^{m-1}frac{1}{c+1} =left[sum_{k=1}^nfrac{1}{k}+sum_{k=1}^mfrac{1}{k}right]-1
        $$







        share|cite|improve this answer














        share|cite|improve this answer



        share|cite|improve this answer








        edited Jan 1 at 3:46

























        answered Jan 1 at 3:34









        InequalitiesEverywhereInequalitiesEverywhere

        1313




        1313








        • 1




          $begingroup$
          As an aside, all of the $approx$'s and the $lesssim$ can be replaced with $leq$'s. So this argument is fully rigorous, but it is also not sharp; one of the other answers does better with the upper bound.
          $endgroup$
          – InequalitiesEverywhere
          Jan 1 at 4:08














        • 1




          $begingroup$
          As an aside, all of the $approx$'s and the $lesssim$ can be replaced with $leq$'s. So this argument is fully rigorous, but it is also not sharp; one of the other answers does better with the upper bound.
          $endgroup$
          – InequalitiesEverywhere
          Jan 1 at 4:08








        1




        1




        $begingroup$
        As an aside, all of the $approx$'s and the $lesssim$ can be replaced with $leq$'s. So this argument is fully rigorous, but it is also not sharp; one of the other answers does better with the upper bound.
        $endgroup$
        – InequalitiesEverywhere
        Jan 1 at 4:08




        $begingroup$
        As an aside, all of the $approx$'s and the $lesssim$ can be replaced with $leq$'s. So this argument is fully rigorous, but it is also not sharp; one of the other answers does better with the upper bound.
        $endgroup$
        – InequalitiesEverywhere
        Jan 1 at 4:08











        4












        $begingroup$

        Let $$S = sum_{k=1}^n frac{1}k + sum_{k=1}^m frac{1}k.$$
        Note that we can write $sum_{k=1}^{nm} frac{1}k$ as
        $$begin{align*}
        1 + cdots + frac{1}n \
        frac1{n+1} + cdots + frac{1}{2n} \
        cdots \
        frac{1}{n(m-1)} + cdots + frac{1}{nm}
        end{align*} $$

        Label the rows of the above table from $1$ upto $m$. Note that all the numbers in the $j$th row are $ge frac{1}{jn}$ so the sum of the numbers in the $j$th row is at least $n cdot frac{1}{jn} = frac{1}j$. Summing this from $j=2$ to $m$ gives us
        $$ sum_{k=1}^{nm} frac{1}k ge left(sum_{k=1}^n frac{1}k + sum_{j=1}^m frac{1}j right) - 1 = S-1.$$
        Similarly, each entry in the $j$th row is at most $frac{1}{(j-1)n}$ so summing from $j=2$ to $m$ gives
        $$ sum_{k=1}^{nm} frac{1}k le left(sum_{k=1}^n frac{1}k + sum_{j=1}^m frac{1}j right) - frac{1}m = S-frac{1}m.$$
        Therefore,
        $$ frac{1}m le sum_{k=1}^n frac{1}k + sum_{k=1}^m frac{1}k - sum_{k=1}^{nm} frac{1}k le 1. $$






        share|cite|improve this answer









        $endgroup$


















          4












          $begingroup$

          Let $$S = sum_{k=1}^n frac{1}k + sum_{k=1}^m frac{1}k.$$
          Note that we can write $sum_{k=1}^{nm} frac{1}k$ as
          $$begin{align*}
          1 + cdots + frac{1}n \
          frac1{n+1} + cdots + frac{1}{2n} \
          cdots \
          frac{1}{n(m-1)} + cdots + frac{1}{nm}
          end{align*} $$

          Label the rows of the above table from $1$ upto $m$. Note that all the numbers in the $j$th row are $ge frac{1}{jn}$ so the sum of the numbers in the $j$th row is at least $n cdot frac{1}{jn} = frac{1}j$. Summing this from $j=2$ to $m$ gives us
          $$ sum_{k=1}^{nm} frac{1}k ge left(sum_{k=1}^n frac{1}k + sum_{j=1}^m frac{1}j right) - 1 = S-1.$$
          Similarly, each entry in the $j$th row is at most $frac{1}{(j-1)n}$ so summing from $j=2$ to $m$ gives
          $$ sum_{k=1}^{nm} frac{1}k le left(sum_{k=1}^n frac{1}k + sum_{j=1}^m frac{1}j right) - frac{1}m = S-frac{1}m.$$
          Therefore,
          $$ frac{1}m le sum_{k=1}^n frac{1}k + sum_{k=1}^m frac{1}k - sum_{k=1}^{nm} frac{1}k le 1. $$






          share|cite|improve this answer









          $endgroup$
















            4












            4








            4





            $begingroup$

            Let $$S = sum_{k=1}^n frac{1}k + sum_{k=1}^m frac{1}k.$$
            Note that we can write $sum_{k=1}^{nm} frac{1}k$ as
            $$begin{align*}
            1 + cdots + frac{1}n \
            frac1{n+1} + cdots + frac{1}{2n} \
            cdots \
            frac{1}{n(m-1)} + cdots + frac{1}{nm}
            end{align*} $$

            Label the rows of the above table from $1$ upto $m$. Note that all the numbers in the $j$th row are $ge frac{1}{jn}$ so the sum of the numbers in the $j$th row is at least $n cdot frac{1}{jn} = frac{1}j$. Summing this from $j=2$ to $m$ gives us
            $$ sum_{k=1}^{nm} frac{1}k ge left(sum_{k=1}^n frac{1}k + sum_{j=1}^m frac{1}j right) - 1 = S-1.$$
            Similarly, each entry in the $j$th row is at most $frac{1}{(j-1)n}$ so summing from $j=2$ to $m$ gives
            $$ sum_{k=1}^{nm} frac{1}k le left(sum_{k=1}^n frac{1}k + sum_{j=1}^m frac{1}j right) - frac{1}m = S-frac{1}m.$$
            Therefore,
            $$ frac{1}m le sum_{k=1}^n frac{1}k + sum_{k=1}^m frac{1}k - sum_{k=1}^{nm} frac{1}k le 1. $$






            share|cite|improve this answer









            $endgroup$



            Let $$S = sum_{k=1}^n frac{1}k + sum_{k=1}^m frac{1}k.$$
            Note that we can write $sum_{k=1}^{nm} frac{1}k$ as
            $$begin{align*}
            1 + cdots + frac{1}n \
            frac1{n+1} + cdots + frac{1}{2n} \
            cdots \
            frac{1}{n(m-1)} + cdots + frac{1}{nm}
            end{align*} $$

            Label the rows of the above table from $1$ upto $m$. Note that all the numbers in the $j$th row are $ge frac{1}{jn}$ so the sum of the numbers in the $j$th row is at least $n cdot frac{1}{jn} = frac{1}j$. Summing this from $j=2$ to $m$ gives us
            $$ sum_{k=1}^{nm} frac{1}k ge left(sum_{k=1}^n frac{1}k + sum_{j=1}^m frac{1}j right) - 1 = S-1.$$
            Similarly, each entry in the $j$th row is at most $frac{1}{(j-1)n}$ so summing from $j=2$ to $m$ gives
            $$ sum_{k=1}^{nm} frac{1}k le left(sum_{k=1}^n frac{1}k + sum_{j=1}^m frac{1}j right) - frac{1}m = S-frac{1}m.$$
            Therefore,
            $$ frac{1}m le sum_{k=1}^n frac{1}k + sum_{k=1}^m frac{1}k - sum_{k=1}^{nm} frac{1}k le 1. $$







            share|cite|improve this answer












            share|cite|improve this answer



            share|cite|improve this answer










            answered Jan 1 at 3:41









            Sandeep SilwalSandeep Silwal

            5,84311236




            5,84311236























                2












                $begingroup$

                Avoiding $log$s we still have $H_N=sum_{k=1}^{N}frac{1}{k}=int_{0}^{1}frac{x^N-1}{x-1}$ and



                $$ H_{NM}-H_{N}-H_{M} = int_{0}^{1}frac{(x^N-1)(x^M-1)}{x-1},dxleq 0, $$
                whose absolute value can be bounded (due to the Cauchy-Schwarz inequality) by
                $$ sqrt{int_{0}^{1}frac{(1-x^N)^2}{1-x},dx int_{0}^{1}frac{(1-x^M)^2}{1-x},dx}=sqrt{(2H_N-H_{2N})(2H_M-H_{2M})}. $$
                On the other hand
                $$ H_{2N}-H_N = sum_{k=1}^{2N}frac{(-1)^{k+1}}{k} $$
                is a partial sum for a convergent (to $log 2$) series, hence
                $$ H_N+H_M-sqrt{H_N H_M}leq H_{NM} leq H_N+H_M $$
                holds for any large $N,M$.






                share|cite|improve this answer









                $endgroup$


















                  2












                  $begingroup$

                  Avoiding $log$s we still have $H_N=sum_{k=1}^{N}frac{1}{k}=int_{0}^{1}frac{x^N-1}{x-1}$ and



                  $$ H_{NM}-H_{N}-H_{M} = int_{0}^{1}frac{(x^N-1)(x^M-1)}{x-1},dxleq 0, $$
                  whose absolute value can be bounded (due to the Cauchy-Schwarz inequality) by
                  $$ sqrt{int_{0}^{1}frac{(1-x^N)^2}{1-x},dx int_{0}^{1}frac{(1-x^M)^2}{1-x},dx}=sqrt{(2H_N-H_{2N})(2H_M-H_{2M})}. $$
                  On the other hand
                  $$ H_{2N}-H_N = sum_{k=1}^{2N}frac{(-1)^{k+1}}{k} $$
                  is a partial sum for a convergent (to $log 2$) series, hence
                  $$ H_N+H_M-sqrt{H_N H_M}leq H_{NM} leq H_N+H_M $$
                  holds for any large $N,M$.






                  share|cite|improve this answer









                  $endgroup$
















                    2












                    2








                    2





                    $begingroup$

                    Avoiding $log$s we still have $H_N=sum_{k=1}^{N}frac{1}{k}=int_{0}^{1}frac{x^N-1}{x-1}$ and



                    $$ H_{NM}-H_{N}-H_{M} = int_{0}^{1}frac{(x^N-1)(x^M-1)}{x-1},dxleq 0, $$
                    whose absolute value can be bounded (due to the Cauchy-Schwarz inequality) by
                    $$ sqrt{int_{0}^{1}frac{(1-x^N)^2}{1-x},dx int_{0}^{1}frac{(1-x^M)^2}{1-x},dx}=sqrt{(2H_N-H_{2N})(2H_M-H_{2M})}. $$
                    On the other hand
                    $$ H_{2N}-H_N = sum_{k=1}^{2N}frac{(-1)^{k+1}}{k} $$
                    is a partial sum for a convergent (to $log 2$) series, hence
                    $$ H_N+H_M-sqrt{H_N H_M}leq H_{NM} leq H_N+H_M $$
                    holds for any large $N,M$.






                    share|cite|improve this answer









                    $endgroup$



                    Avoiding $log$s we still have $H_N=sum_{k=1}^{N}frac{1}{k}=int_{0}^{1}frac{x^N-1}{x-1}$ and



                    $$ H_{NM}-H_{N}-H_{M} = int_{0}^{1}frac{(x^N-1)(x^M-1)}{x-1},dxleq 0, $$
                    whose absolute value can be bounded (due to the Cauchy-Schwarz inequality) by
                    $$ sqrt{int_{0}^{1}frac{(1-x^N)^2}{1-x},dx int_{0}^{1}frac{(1-x^M)^2}{1-x},dx}=sqrt{(2H_N-H_{2N})(2H_M-H_{2M})}. $$
                    On the other hand
                    $$ H_{2N}-H_N = sum_{k=1}^{2N}frac{(-1)^{k+1}}{k} $$
                    is a partial sum for a convergent (to $log 2$) series, hence
                    $$ H_N+H_M-sqrt{H_N H_M}leq H_{NM} leq H_N+H_M $$
                    holds for any large $N,M$.







                    share|cite|improve this answer












                    share|cite|improve this answer



                    share|cite|improve this answer










                    answered Jan 1 at 21:47









                    Jack D'AurizioJack D'Aurizio

                    288k33280660




                    288k33280660























                        1












                        $begingroup$

                        Be the sucession ${c_n}_{ninmathbb{N}}$ as:
                        begin{equation}
                        c_n = sum_{k=1}^n left(frac{1}{k}right) - ln(n)
                        end{equation}

                        We can notice that:
                        begin{eqnarray*}
                        c_{n+1}-c_n &=& left[sum_{k=1}^{n+1} left(frac{1}{k}right) - ln(n+1)right]
                        -left[sum_{k=1}^{n} left(frac{1}{k}right) - ln(n)right] \
                        c_{n+1}-c_n &=& frac{1}{n+1} - ln(n+1) + ln(n) \
                        c_{n+1}-c_n &=& frac{1}{n+1} - left[ln(n+1) - ln(n)right] \
                        c_{n+1}-c_n &=& frac{1}{n+1} - int_{n}^{n+1} frac{1}{x} dx \
                        end{eqnarray*}

                        But if we graph $displaystyle y = frac{1}{x}$ and we draw rectangles as in the figure:



                        Figure



                        We notice that the area of the rectangle between $x=n$ and $x=n+1$ is $displaystylefrac{1}{n+1}$ and it is less than $displaystyleint_{n}^{n+1} frac{1}{x} dx$. Then $forall nin mathbb{N}$:



                        begin{equation}
                        c_{n+1}-c_n = frac{1}{n+1} - int_{n}^{n+1} frac{1}{x} dx < 0
                        end{equation}



                        Thus, the sucession ${c_n}_{ninmathbb{N}}$ is monotonically decreasing and $c_{n+1} < c_n < cdots < c_2 < c_1 = 1$, then $lim_{ntoinfty}c_n=gamma$ exists and it is less than 1, it should be noted that $lim_{ntoinfty}c_n$ is the Euler-Mascheroni constant $gamma$.



                        Be the sucession $leftlbrace b_n rightrbrace_{ninmathbb{N}}$ such that $b_n=nm$ with $minmathbb{N}$, the sucession $c_n$ and its subsucession $c_{b_n}$ converge to the same number:
                        begin{eqnarray}
                        lim_{ntoinfty}c_{b_n} &=& lim_{ntoinfty}c_n \
                        lim_{ntoinfty}sum_{k=1}^{nm} left(frac{1}{k}right) - ln(nm) &=& lim_{ntoinfty}sum_{k=1}^n left(frac{1}{k}right) - ln(n) \
                        lim_{ntoinfty}sum_{k=1}^{nm} left(frac{1}{k}right) - ln(n) - ln(m) &=& lim_{ntoinfty}sum_{k=1}^n left(frac{1}{k}right) - ln(n) \
                        lim_{ntoinfty}sum_{k=1}^{nm} left(frac{1}{k}right) -sum_{k=1}^n left(frac{1}{k}right) &=& ln(m) \
                        lim_{ntoinfty}sum_{k=1}^{nm} left(frac{1}{k}right) -sum_{k=1}^n left(frac{1}{k}right) -sum_{k=1}^{m} left(frac{1}{k}right) &=& ln(m) -sum_{k=1}^{m} left(frac{1}{k}right) \
                        lim_{mtoinfty}lim_{ntoinfty}sum_{k=1}^{nm} left(frac{1}{k}right) -sum_{k=1}^n left(frac{1}{k}right) -sum_{k=1}^{m} left(frac{1}{k}right)
                        &=&
                        lim_{mtoinfty}ln(m) -sum_{k=1}^{m} left(frac{1}{k}right) \
                        lim_{mtoinfty}lim_{ntoinfty}sum_{k=1}^{nm} left(frac{1}{k}right) -sum_{k=1}^n left(frac{1}{k}right) -sum_{k=1}^{m} left(frac{1}{k}right)
                        &=&
                        gamma < 1
                        end{eqnarray}



                        Then for $n,m$ very big:
                        begin{eqnarray}
                        sum_{k=1}^{nm} left(frac{1}{k}right) -sum_{k=1}^n left(frac{1}{k}right) -sum_{k=1}^{m} left(frac{1}{k}right) &approx& gamma \
                        sum_{k=1}^{nm} left(frac{1}{k}right) &approx& gamma + sum_{k=1}^n left(frac{1}{k}right) + sum_{k=1}^{m} left(frac{1}{k}right) \
                        sum_{k=1}^{nm} left(frac{1}{k}right) &approx& sum_{k=1}^n left(frac{1}{k}right) + sum_{k=1}^{m} left(frac{1}{k}right)
                        end{eqnarray}

                        Because $displaystyle gamma << sum_{k=1}^n left(frac{1}{k}right) + sum_{k=1}^{m} left(frac{1}{k}right)$.
                        Part of this demonstration is from the book "Euler: The master of us all" [1].





                        References:
                        Dunham, William, Euler: The master of us all, The Dolciani Mathematical Expositions. 22. Washington, DC: The Mathematical Association of America. xxviii, 185 p. (1999). ZBL0951.01012.






                        share|cite|improve this answer











                        $endgroup$


















                          1












                          $begingroup$

                          Be the sucession ${c_n}_{ninmathbb{N}}$ as:
                          begin{equation}
                          c_n = sum_{k=1}^n left(frac{1}{k}right) - ln(n)
                          end{equation}

                          We can notice that:
                          begin{eqnarray*}
                          c_{n+1}-c_n &=& left[sum_{k=1}^{n+1} left(frac{1}{k}right) - ln(n+1)right]
                          -left[sum_{k=1}^{n} left(frac{1}{k}right) - ln(n)right] \
                          c_{n+1}-c_n &=& frac{1}{n+1} - ln(n+1) + ln(n) \
                          c_{n+1}-c_n &=& frac{1}{n+1} - left[ln(n+1) - ln(n)right] \
                          c_{n+1}-c_n &=& frac{1}{n+1} - int_{n}^{n+1} frac{1}{x} dx \
                          end{eqnarray*}

                          But if we graph $displaystyle y = frac{1}{x}$ and we draw rectangles as in the figure:



                          Figure



                          We notice that the area of the rectangle between $x=n$ and $x=n+1$ is $displaystylefrac{1}{n+1}$ and it is less than $displaystyleint_{n}^{n+1} frac{1}{x} dx$. Then $forall nin mathbb{N}$:



                          begin{equation}
                          c_{n+1}-c_n = frac{1}{n+1} - int_{n}^{n+1} frac{1}{x} dx < 0
                          end{equation}



                          Thus, the sucession ${c_n}_{ninmathbb{N}}$ is monotonically decreasing and $c_{n+1} < c_n < cdots < c_2 < c_1 = 1$, then $lim_{ntoinfty}c_n=gamma$ exists and it is less than 1, it should be noted that $lim_{ntoinfty}c_n$ is the Euler-Mascheroni constant $gamma$.



                          Be the sucession $leftlbrace b_n rightrbrace_{ninmathbb{N}}$ such that $b_n=nm$ with $minmathbb{N}$, the sucession $c_n$ and its subsucession $c_{b_n}$ converge to the same number:
                          begin{eqnarray}
                          lim_{ntoinfty}c_{b_n} &=& lim_{ntoinfty}c_n \
                          lim_{ntoinfty}sum_{k=1}^{nm} left(frac{1}{k}right) - ln(nm) &=& lim_{ntoinfty}sum_{k=1}^n left(frac{1}{k}right) - ln(n) \
                          lim_{ntoinfty}sum_{k=1}^{nm} left(frac{1}{k}right) - ln(n) - ln(m) &=& lim_{ntoinfty}sum_{k=1}^n left(frac{1}{k}right) - ln(n) \
                          lim_{ntoinfty}sum_{k=1}^{nm} left(frac{1}{k}right) -sum_{k=1}^n left(frac{1}{k}right) &=& ln(m) \
                          lim_{ntoinfty}sum_{k=1}^{nm} left(frac{1}{k}right) -sum_{k=1}^n left(frac{1}{k}right) -sum_{k=1}^{m} left(frac{1}{k}right) &=& ln(m) -sum_{k=1}^{m} left(frac{1}{k}right) \
                          lim_{mtoinfty}lim_{ntoinfty}sum_{k=1}^{nm} left(frac{1}{k}right) -sum_{k=1}^n left(frac{1}{k}right) -sum_{k=1}^{m} left(frac{1}{k}right)
                          &=&
                          lim_{mtoinfty}ln(m) -sum_{k=1}^{m} left(frac{1}{k}right) \
                          lim_{mtoinfty}lim_{ntoinfty}sum_{k=1}^{nm} left(frac{1}{k}right) -sum_{k=1}^n left(frac{1}{k}right) -sum_{k=1}^{m} left(frac{1}{k}right)
                          &=&
                          gamma < 1
                          end{eqnarray}



                          Then for $n,m$ very big:
                          begin{eqnarray}
                          sum_{k=1}^{nm} left(frac{1}{k}right) -sum_{k=1}^n left(frac{1}{k}right) -sum_{k=1}^{m} left(frac{1}{k}right) &approx& gamma \
                          sum_{k=1}^{nm} left(frac{1}{k}right) &approx& gamma + sum_{k=1}^n left(frac{1}{k}right) + sum_{k=1}^{m} left(frac{1}{k}right) \
                          sum_{k=1}^{nm} left(frac{1}{k}right) &approx& sum_{k=1}^n left(frac{1}{k}right) + sum_{k=1}^{m} left(frac{1}{k}right)
                          end{eqnarray}

                          Because $displaystyle gamma << sum_{k=1}^n left(frac{1}{k}right) + sum_{k=1}^{m} left(frac{1}{k}right)$.
                          Part of this demonstration is from the book "Euler: The master of us all" [1].





                          References:
                          Dunham, William, Euler: The master of us all, The Dolciani Mathematical Expositions. 22. Washington, DC: The Mathematical Association of America. xxviii, 185 p. (1999). ZBL0951.01012.






                          share|cite|improve this answer











                          $endgroup$
















                            1












                            1








                            1





                            $begingroup$

                            Be the sucession ${c_n}_{ninmathbb{N}}$ as:
                            begin{equation}
                            c_n = sum_{k=1}^n left(frac{1}{k}right) - ln(n)
                            end{equation}

                            We can notice that:
                            begin{eqnarray*}
                            c_{n+1}-c_n &=& left[sum_{k=1}^{n+1} left(frac{1}{k}right) - ln(n+1)right]
                            -left[sum_{k=1}^{n} left(frac{1}{k}right) - ln(n)right] \
                            c_{n+1}-c_n &=& frac{1}{n+1} - ln(n+1) + ln(n) \
                            c_{n+1}-c_n &=& frac{1}{n+1} - left[ln(n+1) - ln(n)right] \
                            c_{n+1}-c_n &=& frac{1}{n+1} - int_{n}^{n+1} frac{1}{x} dx \
                            end{eqnarray*}

                            But if we graph $displaystyle y = frac{1}{x}$ and we draw rectangles as in the figure:



                            Figure



                            We notice that the area of the rectangle between $x=n$ and $x=n+1$ is $displaystylefrac{1}{n+1}$ and it is less than $displaystyleint_{n}^{n+1} frac{1}{x} dx$. Then $forall nin mathbb{N}$:



                            begin{equation}
                            c_{n+1}-c_n = frac{1}{n+1} - int_{n}^{n+1} frac{1}{x} dx < 0
                            end{equation}



                            Thus, the sucession ${c_n}_{ninmathbb{N}}$ is monotonically decreasing and $c_{n+1} < c_n < cdots < c_2 < c_1 = 1$, then $lim_{ntoinfty}c_n=gamma$ exists and it is less than 1, it should be noted that $lim_{ntoinfty}c_n$ is the Euler-Mascheroni constant $gamma$.



                            Be the sucession $leftlbrace b_n rightrbrace_{ninmathbb{N}}$ such that $b_n=nm$ with $minmathbb{N}$, the sucession $c_n$ and its subsucession $c_{b_n}$ converge to the same number:
                            begin{eqnarray}
                            lim_{ntoinfty}c_{b_n} &=& lim_{ntoinfty}c_n \
                            lim_{ntoinfty}sum_{k=1}^{nm} left(frac{1}{k}right) - ln(nm) &=& lim_{ntoinfty}sum_{k=1}^n left(frac{1}{k}right) - ln(n) \
                            lim_{ntoinfty}sum_{k=1}^{nm} left(frac{1}{k}right) - ln(n) - ln(m) &=& lim_{ntoinfty}sum_{k=1}^n left(frac{1}{k}right) - ln(n) \
                            lim_{ntoinfty}sum_{k=1}^{nm} left(frac{1}{k}right) -sum_{k=1}^n left(frac{1}{k}right) &=& ln(m) \
                            lim_{ntoinfty}sum_{k=1}^{nm} left(frac{1}{k}right) -sum_{k=1}^n left(frac{1}{k}right) -sum_{k=1}^{m} left(frac{1}{k}right) &=& ln(m) -sum_{k=1}^{m} left(frac{1}{k}right) \
                            lim_{mtoinfty}lim_{ntoinfty}sum_{k=1}^{nm} left(frac{1}{k}right) -sum_{k=1}^n left(frac{1}{k}right) -sum_{k=1}^{m} left(frac{1}{k}right)
                            &=&
                            lim_{mtoinfty}ln(m) -sum_{k=1}^{m} left(frac{1}{k}right) \
                            lim_{mtoinfty}lim_{ntoinfty}sum_{k=1}^{nm} left(frac{1}{k}right) -sum_{k=1}^n left(frac{1}{k}right) -sum_{k=1}^{m} left(frac{1}{k}right)
                            &=&
                            gamma < 1
                            end{eqnarray}



                            Then for $n,m$ very big:
                            begin{eqnarray}
                            sum_{k=1}^{nm} left(frac{1}{k}right) -sum_{k=1}^n left(frac{1}{k}right) -sum_{k=1}^{m} left(frac{1}{k}right) &approx& gamma \
                            sum_{k=1}^{nm} left(frac{1}{k}right) &approx& gamma + sum_{k=1}^n left(frac{1}{k}right) + sum_{k=1}^{m} left(frac{1}{k}right) \
                            sum_{k=1}^{nm} left(frac{1}{k}right) &approx& sum_{k=1}^n left(frac{1}{k}right) + sum_{k=1}^{m} left(frac{1}{k}right)
                            end{eqnarray}

                            Because $displaystyle gamma << sum_{k=1}^n left(frac{1}{k}right) + sum_{k=1}^{m} left(frac{1}{k}right)$.
                            Part of this demonstration is from the book "Euler: The master of us all" [1].





                            References:
                            Dunham, William, Euler: The master of us all, The Dolciani Mathematical Expositions. 22. Washington, DC: The Mathematical Association of America. xxviii, 185 p. (1999). ZBL0951.01012.






                            share|cite|improve this answer











                            $endgroup$



                            Be the sucession ${c_n}_{ninmathbb{N}}$ as:
                            begin{equation}
                            c_n = sum_{k=1}^n left(frac{1}{k}right) - ln(n)
                            end{equation}

                            We can notice that:
                            begin{eqnarray*}
                            c_{n+1}-c_n &=& left[sum_{k=1}^{n+1} left(frac{1}{k}right) - ln(n+1)right]
                            -left[sum_{k=1}^{n} left(frac{1}{k}right) - ln(n)right] \
                            c_{n+1}-c_n &=& frac{1}{n+1} - ln(n+1) + ln(n) \
                            c_{n+1}-c_n &=& frac{1}{n+1} - left[ln(n+1) - ln(n)right] \
                            c_{n+1}-c_n &=& frac{1}{n+1} - int_{n}^{n+1} frac{1}{x} dx \
                            end{eqnarray*}

                            But if we graph $displaystyle y = frac{1}{x}$ and we draw rectangles as in the figure:



                            Figure



                            We notice that the area of the rectangle between $x=n$ and $x=n+1$ is $displaystylefrac{1}{n+1}$ and it is less than $displaystyleint_{n}^{n+1} frac{1}{x} dx$. Then $forall nin mathbb{N}$:



                            begin{equation}
                            c_{n+1}-c_n = frac{1}{n+1} - int_{n}^{n+1} frac{1}{x} dx < 0
                            end{equation}



                            Thus, the sucession ${c_n}_{ninmathbb{N}}$ is monotonically decreasing and $c_{n+1} < c_n < cdots < c_2 < c_1 = 1$, then $lim_{ntoinfty}c_n=gamma$ exists and it is less than 1, it should be noted that $lim_{ntoinfty}c_n$ is the Euler-Mascheroni constant $gamma$.



                            Be the sucession $leftlbrace b_n rightrbrace_{ninmathbb{N}}$ such that $b_n=nm$ with $minmathbb{N}$, the sucession $c_n$ and its subsucession $c_{b_n}$ converge to the same number:
                            begin{eqnarray}
                            lim_{ntoinfty}c_{b_n} &=& lim_{ntoinfty}c_n \
                            lim_{ntoinfty}sum_{k=1}^{nm} left(frac{1}{k}right) - ln(nm) &=& lim_{ntoinfty}sum_{k=1}^n left(frac{1}{k}right) - ln(n) \
                            lim_{ntoinfty}sum_{k=1}^{nm} left(frac{1}{k}right) - ln(n) - ln(m) &=& lim_{ntoinfty}sum_{k=1}^n left(frac{1}{k}right) - ln(n) \
                            lim_{ntoinfty}sum_{k=1}^{nm} left(frac{1}{k}right) -sum_{k=1}^n left(frac{1}{k}right) &=& ln(m) \
                            lim_{ntoinfty}sum_{k=1}^{nm} left(frac{1}{k}right) -sum_{k=1}^n left(frac{1}{k}right) -sum_{k=1}^{m} left(frac{1}{k}right) &=& ln(m) -sum_{k=1}^{m} left(frac{1}{k}right) \
                            lim_{mtoinfty}lim_{ntoinfty}sum_{k=1}^{nm} left(frac{1}{k}right) -sum_{k=1}^n left(frac{1}{k}right) -sum_{k=1}^{m} left(frac{1}{k}right)
                            &=&
                            lim_{mtoinfty}ln(m) -sum_{k=1}^{m} left(frac{1}{k}right) \
                            lim_{mtoinfty}lim_{ntoinfty}sum_{k=1}^{nm} left(frac{1}{k}right) -sum_{k=1}^n left(frac{1}{k}right) -sum_{k=1}^{m} left(frac{1}{k}right)
                            &=&
                            gamma < 1
                            end{eqnarray}



                            Then for $n,m$ very big:
                            begin{eqnarray}
                            sum_{k=1}^{nm} left(frac{1}{k}right) -sum_{k=1}^n left(frac{1}{k}right) -sum_{k=1}^{m} left(frac{1}{k}right) &approx& gamma \
                            sum_{k=1}^{nm} left(frac{1}{k}right) &approx& gamma + sum_{k=1}^n left(frac{1}{k}right) + sum_{k=1}^{m} left(frac{1}{k}right) \
                            sum_{k=1}^{nm} left(frac{1}{k}right) &approx& sum_{k=1}^n left(frac{1}{k}right) + sum_{k=1}^{m} left(frac{1}{k}right)
                            end{eqnarray}

                            Because $displaystyle gamma << sum_{k=1}^n left(frac{1}{k}right) + sum_{k=1}^{m} left(frac{1}{k}right)$.
                            Part of this demonstration is from the book "Euler: The master of us all" [1].





                            References:
                            Dunham, William, Euler: The master of us all, The Dolciani Mathematical Expositions. 22. Washington, DC: The Mathematical Association of America. xxviii, 185 p. (1999). ZBL0951.01012.







                            share|cite|improve this answer














                            share|cite|improve this answer



                            share|cite|improve this answer








                            edited Jan 17 at 20:45

























                            answered Jan 1 at 5:51









                            El PastaEl Pasta

                            46015




                            46015






























                                draft saved

                                draft discarded




















































                                Thanks for contributing an answer to Mathematics Stack Exchange!


                                • Please be sure to answer the question. Provide details and share your research!

                                But avoid



                                • Asking for help, clarification, or responding to other answers.

                                • Making statements based on opinion; back them up with references or personal experience.


                                Use MathJax to format equations. MathJax reference.


                                To learn more, see our tips on writing great answers.




                                draft saved


                                draft discarded














                                StackExchange.ready(
                                function () {
                                StackExchange.openid.initPostLogin('.new-post-login', 'https%3a%2f%2fmath.stackexchange.com%2fquestions%2f3058192%2fshowing-sum-k-1nm-frac1k-approx-sum-k-1n-frac1k-sum%23new-answer', 'question_page');
                                }
                                );

                                Post as a guest















                                Required, but never shown





















































                                Required, but never shown














                                Required, but never shown












                                Required, but never shown







                                Required, but never shown

































                                Required, but never shown














                                Required, but never shown












                                Required, but never shown







                                Required, but never shown







                                Popular posts from this blog

                                Human spaceflight

                                Can not write log (Is /dev/pts mounted?) - openpty in Ubuntu-on-Windows?

                                File:DeusFollowingSea.jpg